Đến nội dung

funcalys nội dung

Có 565 mục bởi funcalys (Tìm giới hạn từ 30-03-2020)



Sắp theo                Sắp xếp  

#531022 Find a strictly increasing function f such that $f'(1)=0$

Đã gửi bởi funcalys on 29-10-2014 - 07:03 trong Mathematics in English

Let $f(x)=\frac{x^3}{3}-x^2+x$
Then its derivative vanishes at x=1 and remains positive elsewhere. 




#517980 Change of variables

Đã gửi bởi funcalys on 06-08-2014 - 14:45 trong Mathematics in English

Introduce a suitable change of variables:

$\iint_{S}f(ax+by+c)dxdy=2\int_{-1}^{1}\sqrt{1-u^2}f(u\sqrt{a^2+b^2 }+c)du$ where S is the unit circle and $a^2+b^2$ is nonzero

(The above excercise was taken from Apostol's Calculus vol. 2/pg. 401)

 

 

 




#503615 Góp ý về việc hỏi bài của các thành viên

Đã gửi bởi funcalys on 02-06-2014 - 19:27 trong Góp ý cho diễn đàn

Chào mọi người,

Em muốn góp ý về việc chỉnh sửa khi một thành viên post bài.

Để nhằm nâng cao sự hiểu bài thì người học phải cần có những bước đầu tự giải bài toán. Và mục đích của chúng ta nên là giải đáp thắc mắc của một thành viên hơn là giải toàn bộ bài của thành viên đó post lên. 

Vậy nên em góp ý là mỗi khi một thành viên gửi bài, nên có lựa chọn ô đánh dấu có phải bài tập hay không, và nếu đánh dấu vào box BÀI TẬP sẽ tự động có tag [Bài tập] trước tên chủ đề và bố cục của ô trả lời sẽ thay đổi như:

1.Bài toán

2.Câu hỏi liên quan

3.Bạn đã giải đến đâu rồi

Tuy có thể mọi người xem cái này là không cần thiết, rằng đôi khi người hỏi cũng nêu những phần trên. Nhưng đó chỉ là số ít, Còn nhiều thành viên cứ post bài lên cho mọi người giải mà không cho người khác xem mình giải đến đâu để được góp ý. Và điều này có thể tăng tích cực hiệu quả của việc học hỏi. Chắc mọi người khi post bài giải cũng đều mong muốn người hỏi chí ít cũng đã thử qua bài toán chứ giải hết thì thật cũng khá ngán ngẩm.

Một trong số những diễn đàn đã áp dụng cái này là physicsforums.com và chắc cũng trên nhiều diễn đàn khoa học nói chung, mọi người đều mong một sự cố gắng ở những người đặt câu hỏi.

"Hỏi một câu chỉ không biết trong chốc lát,không hỏi sẽ không biết cả đời"

:)

 




#503177 $\bigl(\begin{smallmatrix} 0 & b\\ a+b...

Đã gửi bởi funcalys on 01-06-2014 - 09:49 trong Đại số tuyến tính, Hình học giải tích

À xin lỗi bạn, hqua mình vội nên nhầm qua toán tử tuyến tính

Để c/m không gian con thì bn chứng minh nó đóng với phép + và nhân vô hướng

Tức là $u+v\in M$ và $ku\in M,k\in K$

bạn nhầm qua tổng các không gian rồi, ở đây, ta chỉ cần xét vector đơn lẻ (ở đây là ma trận)

Cụ thể:
$\begin{pmatrix}0 &b \\ a+b &0 \end{pmatrix}+\begin{pmatrix} 0&c \\ d+c & 0\end{pmatrix}=\begin{pmatrix}0 &b+c \\ a+d+b+c &0 \end{pmatrix}\in M$
$k\begin{pmatrix}0 &b \\ a+b &0 \end{pmatrix}=\begin{pmatrix}0&kb \\ ka+kb &0 \end{pmatrix}\in M$
Và hiển nhiên $0\in M$
Vậy M là không gian con của $M_{22}(\mathbb{K})$
b/ Bạn viết hơi tối nghĩa:
Ta có
TÍnh độc lập tuyến tính
Giả sử $a,b\neq 0$
$\alpha\begin{pmatrix}0 &b \\ b &0 \end{pmatrix}+\beta\begin{pmatrix}0 &0 \\ c &0 \end{pmatrix}=0\iff \alpha b=0\wedge \alpha b+\beta c=0$
Ta có $\alpha=0\Rightarrow\beta=0$

Vậy 2 vector độc lập tuyến  tính

Mọi vector đều biểu diễn được dưới tổ hợp tuyến tính của 2 vector này:

Điều này dễ thấy

Đến đây bạn kết luận được r.

 




#503106 $\bigl(\begin{smallmatrix} 0 & b\\ a+b...

Đã gửi bởi funcalys on 31-05-2014 - 22:36 trong Đại số tuyến tính, Hình học giải tích

Câu a chứng minh $\alpha A+\beta B\in M$ với $A,B\in M$

Câu b

Một cơ sở cho M là $\left \{ \begin{pmatrix}0 &a \\ a &0 \end{pmatrix},\left (\begin{pmatrix}0 &0 \\ b &0\end{pmatrix}  \right ) \right \}$
(kiểm tra tính độc lập tuyến tính và mọi phần tử M đều biểu diễn được dưới tổ hợp tuyến tính các phần tử trong tập cơ sở)
dimM=2



#502943 Ma trận lũy linh

Đã gửi bởi funcalys on 31-05-2014 - 11:31 trong Đại số tuyến tính, Hình học giải tích

À, phần đó được đề cập trong sách của thầy Nguyễn Hữu Việt Hưng ấy bạn, nằm trước phần dạng chuẩn tắc Jordan.

Ở đây có lẽ bạn không cần dùng đến định nghĩa và biến đổi các ma trận mà chỉ cần lập luận như trên.

Ta có định lí rằng nếu $A\in M_n(\mathbb{R})$ thì $A\approx D$ với D là một ma trận tam giác. Do A và D đồng dạng nên $\lambda_A=\lambda_D=0$ hay các phần tử trên đường chéo của ma trận này đều bằng 0.

0




#502900 Ma trận lũy linh

Đã gửi bởi funcalys on 31-05-2014 - 09:03 trong Đại số tuyến tính, Hình học giải tích

Do A lũy linh nên ta có phân tích V thành tổng trực tiếp các không gian con cyclic đối với toán tử tuyến tính biểu diễn A:

Vậy A, đối với một phân tích nào đó, có dạng:

$\begin{pmatrix}M_1 &  &  &0 \\  &M_2  &  & \\ &  &...  & \\  0& &  &M_n \end{pmatrix}$
Với khối $M_j$ có dạng $\begin{pmatrix}0 &  &  &0 \\ 1 & 0 &  & \\  & ... &...  & \\ 0 &  &1  &0 \end{pmatrix}$
Và tổng các cấp của các khối bằng n
Vậy ta dễ có được $\det(A-\lambda I_n)=(-1)^n\lambda^n$
Do A lũy linh nên A chỉ có giá trị riêng duy nhất là 0, mà ta có mọi ma trận thực đều đồng dạng với một ma trận tam giác nào đó. Để 2 ma trận này đồng dạng bắt buộc D phải có giá trị riêng bằng 0 hay các phần tử trên đường chéo chính của nó bằng 0.



#501277 ma trận liên hợp

Đã gửi bởi funcalys on 24-05-2014 - 20:03 trong Đại số tuyến tính, Hình học giải tích

$\begin{bmatrix}\widehat{A_{11}} &-\widehat{A_{12}}  &\widehat{A_{13}} \\ -\widehat{A_{21}} & \widehat{A_{22}} & -\widehat{A_{23}}\\ \widehat{A_{31}} & -\widehat{A_{33}} & \widehat{A_{33}}\end{bmatrix}$
Bạn cứ áp dụng ct trên, với chỉ số đầu là dòng, chỉ số sau là cột loại bỏ.
Đáp án:
$\begin{bmatrix}1 &-10  &7 \\ 1 & 4 &-3 \\ -1 & 2 & -1\end{bmatrix}$

 




#499731 Chứng minh $x_1+x_2+x_3+x_4+x_5=0$

Đã gửi bởi funcalys on 18-05-2014 - 05:41 trong Tổ hợp - Xác suất và thống kê - Số phức

Do $x_1,...,x_5$ là nghiệm của phương trình $x^5-1=0$, áp dụng Viète ta có được $\sum x_j =0$

Áp dụng tương tự nếu ta mở rộng lên căn bậc n của đơn vị.




#497694 Tính modun của số phức

Đã gửi bởi funcalys on 07-05-2014 - 20:35 trong Đại số tuyến tính, Hình học giải tích

Do $\left | \frac{a}{b} \right |=\frac{\left | a \right |}{\left | b \right |}$ và ở có công thức module số phức  $\left | a+jb \right |=\sqrt{a^2+b^2}$




#497406 Tính modun của số phức

Đã gửi bởi funcalys on 06-05-2014 - 08:10 trong Đại số tuyến tính, Hình học giải tích

Bạn lấy module phần dưới là được

$\left | \frac{a}{c+jb} \right |=\frac{\left | a \right |}{\sqrt{b^2+c^2}}$




#496434 $\int_{-\infty }^{0}xe^{2x}dx$

Đã gửi bởi funcalys on 01-05-2014 - 19:34 trong Tích phân - Nguyên hàm

Sử dụng L'hôpital là đc

$\lim te^{2t}=\frac{1}{2}\lim \frac{2t}{e^{-2t}}=\frac{1}{2}\lim \frac{\frac{\partial }{\partial 2t}2t}{\frac{\partial }{\partial 2t} e^{-2t}}=\lim_{n\to -\infty}\frac{-1}{2}e^{2t}=0$




#494027 Không gian con

Đã gửi bởi funcalys on 20-04-2014 - 07:54 trong Đại số tuyến tính, Hình học giải tích

Lấy ánh xạ đồng nhất thì được gì bạn ?




#493862 Xác định $\alpha$ để dãy $f_n(x)=n^\alpha x e^{...

Đã gửi bởi funcalys on 19-04-2014 - 12:13 trong Giải tích

Bên tích phân Lebesgue còn 2 tiêu chuẩn nữa để chuyển giới hạn dưới dấu tích phân (định lí hội tụ đơn điệu và định lí hội tụ bị chặn, tất nhiên là điều kiện không cần mạnh như hội tụ đều) nhưng do mình nghĩ ở đây bạn đang nói tích phân Riemann nên mình nghĩ chỉ sử dụng tiêu chuẩn là dãy hàm hội tụ đều.




#493690 Xác định $\alpha$ để dãy $f_n(x)=n^\alpha x e^{...

Đã gửi bởi funcalys on 18-04-2014 - 12:57 trong Giải tích

Nghĩa là $\lim \int f_n=\int \lim f_n$

Ta có được điều này khi $f_n$ hội tụ đều




#492834 đại số

Đã gửi bởi funcalys on 14-04-2014 - 10:39 trong Tài liệu và chuyên đề Đại số đại cương

Do $k\in \mathbb{N}^*$ nên ta có thể đặt $b=a/k$ để quy bài toán về việc chứng minh phần nguyên của b là duy nhất.

Thật thế, giả sử

$m=\left \lfloor b \right \rfloor=n$

($m\neq n$)

Theo định nghĩa, ta có b thỏa mãn 2 bất đẳng thức;

$m\leq b<m+1$

$n\leq b<n+1$

Không mất tính tổng quát, giả sử

$n<m$

Khi đó

$n+1\leq m$

Giả sử ngược lại nếu $n+1>m$

Ta sẽ có $n+1\geq m+1$ điều này là mâu thuẫn do bất đẳng thức đầu tiên là ngặt.

Vậy $b>m\geq n+1$

Hay $a\geq k(n+1)$

Suy ra mâu thuẫn, từ đây ta có đpcm.

 




#492608 hỏi về vành và vành con

Đã gửi bởi funcalys on 13-04-2014 - 09:33 trong Tài liệu và chuyên đề Đại số đại cương

Vành con là một tập con của vành đang xét sao cho nó vẫn giữ cấu trúc của một vành nên bản thân nó là một vành với các phép toán của vành đang xét.




#491200 hỏi về vành và vành con

Đã gửi bởi funcalys on 07-04-2014 - 05:55 trong Tài liệu và chuyên đề Đại số đại cương

Giống như 1 cách để cm 1 kgvt là chứng minh nó là một kgvt con th.




#491177 hỏi về vành và vành con

Đã gửi bởi funcalys on 06-04-2014 - 22:04 trong Tài liệu và chuyên đề Đại số đại cương

Đúng.




#491176 không gian tổng, không gian giao

Đã gửi bởi funcalys on 06-04-2014 - 22:03 trong Đại số tuyến tính, Hình học giải tích

Bạn chưa rõ phần nào cứ nêu câu hỏi cụ thể.




#490119 căn nguyên thuỷ bậc n của đơn vị

Đã gửi bởi funcalys on 02-04-2014 - 05:08 trong Đại số đại cương

Căn nguyên thủy bậc 3 của đơn vị là $e^{i\frac{2k\pi}{3}}$ th bạn, chúng là nghiệm của phương trình $x^3-1=0$, vậy thì nghiệm của phương trình sẽ có dạng $\sqrt[3]{5}e^{i\frac{2k\pi}{3}},k=1,2,3...$




#489972 căn nguyên thuỷ bậc n của đơn vị

Đã gửi bởi funcalys on 01-04-2014 - 13:25 trong Đại số đại cương

Bạn giải nghiệm của đa thức ra, được $F=\mathbb{Q}(\sqrt[3]{5},\frac{-1}{2}+i \frac{\sqrt 3}{2})$

Ta có đa thức tối tiểu của $\frac{-1}{2}+i \frac{\sqrt 3}{2}$ là $x^2+x+1$

đa thức tối tiểu của $\sqrt[3]{5}$ là $x^3-5$

Áp dụng quy tắc tháp, ta có;

$\left [ \mathbb{Q}(\sqrt[3]{5},\frac{-1}{2}+i \frac{\sqrt 3}{2}):\mathbb{Q} \right ]=\left [ \mathbb{Q}(\sqrt[3]{5},\frac{-1}{2}+i \frac{\sqrt 3}{2}):\mathbb{Q}(\sqrt[3]{5}) \right ]\left [\mathbb{Q}(\sqrt[3]{5}):\mathbb{Q}  \right ]=6$




#488761 Độ đo lebesgue

Đã gửi bởi funcalys on 25-03-2014 - 21:11 trong Tôpô

Xem cách xây dựng tập Vitali




#485789 Tìm sách về độ đo và tích phân Lebesgue

Đã gửi bởi funcalys on 04-03-2014 - 12:57 trong Tài liệu, chuyên đề Toán cao cấp

Bắt đầu thì nên đọc Elements of Measure and Integration (k biết mình nhớ đúng tên k) của Bartles

Nâng cao theo mình thấy có cuốn của Folland và Royden

Còn để tham khảo là chính thì có bộ 2 volumes của Bogachev

Những cuốn n` đều có trên libgen hoặc bookfi :)




#483813 Giải chi tiết giùm mình với !

Đã gửi bởi funcalys on 18-02-2014 - 12:20 trong Tài liệu và chuyên đề Đại số đại cương

Với I là một ideal của vành giao hoán R, ta có: R/I là trường khi và chỉ khi I tối đại.

Vậy ta sẽ chứng minh $p\mathbb{Z}$ là một ideal tối đại.

Giả sử $p\mathbb{Z}$ không tối đại, tức tồn tại 1 ideal $I\neq \mathbb{Z}$: $p\mathbb{Z}\subset I$ 

Lấy $x\in I\setminus p\mathbb{Z}$ ta có: gcd(p,x)=1 (do p không là ước của x)

Áp dụng định lí Bézout, ta tìm được $a,b\in \mathbb{Z}$ sao cho:

ap+bx=1

Ta có $ap+bx\in I$ nên $1\in I$, vậy $I=\mathbb{Z}$, trái với giả thuyết, vậy $p\mathbb{Z}$ tối đại.